8 votos

Grothendieck universo consistencia

Es ZFC con la existencia de Grothendieck del universo (variante: Grothendieck del universo que contiene cada conjunto dado) es demostrable en ZFC a ser equiconsistent con ZFC?

Si no, ¿qué otra cosa puede ser equiconsistent? (Y en el que los formalistas puede ser demostrado?)

5voto

zyx Puntos 20965

Este asunto fue discutido mucho sobre MathOverflow. Los enlaces a continuación y en la página de la Wikipedia debe responder a la pregunta y a la que aparece en los comentarios.

http://mathoverflow.net/questions/35746/inaccessible-cardinals-and-andrew-wiless-proof

http://mathoverflow.net/questions/12804/large-cardinal-axioms-and-grothendieck-universes

http://en.wikipedia.org/wiki/Grothendieck_universe

i-Ciencias.com

I-Ciencias es una comunidad de estudiantes y amantes de la ciencia en la que puedes resolver tus problemas y dudas.
Puedes consultar las preguntas de otros usuarios, hacer tus propias preguntas o resolver las de los demás.

Powered by:

X